Đến nội dung

Nguyenhuyen_AG

Nguyenhuyen_AG

Đăng ký: 09-09-2010
Offline Đăng nhập: 10-01-2019 - 16:22
****-

#691259 Chứng minh $(a^2 +2)(b^2+2)(c^2+2) \ge 3(a+b+c)^2$

Gửi bởi Nguyenhuyen_AG trong 21-08-2017 - 23:47

Anh ơi cho em hỏi là anh dùng kĩ thuật gì để ra cái này ạ

 

Anh dùng hệ số bất định.




#691236 BẤT ĐẲNG THỨC

Gửi bởi Nguyenhuyen_AG trong 21-08-2017 - 20:49

Cho a,b,c > 0. Chứng minh rằng:

                                             $a^{3}+b^{3}+c^{3}\geq ab^{2}+bc^{2}+ca^{2}$

 

Giả sử $c=\min\{a,b,c\}$ khi đó

\[a^{3}+b^{3}+c^{3}-(ab^{2}+bc^{2}+ca^{2}) = (a+b)(a-b)^2+(c+a)(a-c)(b-c) \geqslant 0.\]




#691166 Chứng minh $(a^2 +2)(b^2+2)(c^2+2) \ge 3(a+b+c)^2$

Gửi bởi Nguyenhuyen_AG trong 20-08-2017 - 20:59

Cho $a,b,c >0$. Chứng minh:

$(a^2 +2)(b^2+2)(c^2+2) \ge 3(a+b+c)^2$

 

Ta có

\[\text{Vế trái  -  Vế phải} = \frac{\displaystyle 3\sum (c^2+5)(ab-1)^2 + \sum (a+b-2c)^2 + 3\left(\sum ab -3\right)^2}{9}.\]




#690911 Chứng minh $\sum {\frac{{a{b^2}}...

Gửi bởi Nguyenhuyen_AG trong 18-08-2017 - 18:44

Có một cách đơn giản hớn là chứng minh

\[\sum \frac{x}{x^3+1} \leqslant \frac34 \sum \frac{x+1}{x^2+x+1} \leqslant \frac32.\]




#690707 xab+ybc+zca

Gửi bởi Nguyenhuyen_AG trong 16-08-2017 - 22:41

có ai biết cách làm dạng này ko?

a)Cho a+b+c=1. Tìm max A= xab+ ybc +zca (x,y,z là các stn)

b ) Cho ab+ bc+ca=1 .tìm min B= xa^2+ yb^2 +zc^2 (x,y,z là các stn)

Tks nh :))

Câu a dùng tam thức, câu b dùng Cauchy-Schwarz.




#690419 $\sum \frac{a}{a+b}+\frac{abc...

Gửi bởi Nguyenhuyen_AG trong 13-08-2017 - 13:23

Còn link không anh ? 

 

Link trên VMF anh thử tìm nhưng không ra: http://k2pi.net.vn/s...ead.php?p=78215




#690418 $1+\frac{1}{x^2}$ ≥ $cosA+x(cosB+cosC...

Gửi bởi Nguyenhuyen_AG trong 13-08-2017 - 13:05

 

Cho tam giác ABC là một tam giác bất kì. Chứng minh rằng với mọi số x ta đều có

 

$1+\frac{1}{2}x^2$ ≥ $cosA+x(cosB+cosC)$

 

Xem hiệu hai vế là một tam thức bậc hai theo $x$ với hệ số của $x^2$ dương. Để chứng minh tam thức này không âm ta chỉ cần chứng minh biệt thức $\Delta$ của nó luôn $\leqslant 0$ là được.

 

Thật vậy

\[\begin{aligned}\Delta & = (\cos B + \cos C)^2 + 2\cos A - 2 \\&= \frac14\left(\frac{c^2+a^2-b^2}{ca}+\frac{a^2+b^2-c^2}{ab}\right)^2 + \frac{b^2+c^2-a^2}{bc}-2 \\&= -\frac{(b-c)^2\left[2(a^2b^2+b^2c^2+c^2a^2)-a^4-b^4-c^4\right] }{4a^2b^2c^2} \leqslant 0.\end{aligned}\]

Ta có điều phải chứng minh.




#690337 Tìm giá trị lớn nhất và nhỏ nhất của biểu thức E = 2xy - y^2

Gửi bởi Nguyenhuyen_AG trong 12-08-2017 - 18:39

Còn GTLN ạ...

 

Ta có

\[2xy-y^2 = \frac14(x^2+2xy+5y^2-8)-\frac14(x-3y)^2+2 \leqslant 2.\]

Đẳng thức xảy ra khi $x=3y.$




#690336 $\sum \frac{a}{a+b}+\frac{abc...

Gửi bởi Nguyenhuyen_AG trong 12-08-2017 - 18:27

Lời giải của hanguyen445 sai vì bất đẳng thức $(*)$ không đúng. Bài này có thể chứng minh bằng Cauchy-Schwarz lúc trước có một bạn đã đăng lên diễn đàn và mình cũng có đăng một bài mạnh hơn của nó.




#690248 Cho $a,b,c>0$ thỏa $ab+bc+ca=\frac{1}{...

Gửi bởi Nguyenhuyen_AG trong 11-08-2017 - 20:34

Cho $a,b,c>0$ thỏa $ab+bc+ca=\frac{1}{3}$. CM: $\sum \frac{1}{a^2-bc+1} \le 3$

 

Bài này là China TST 2005.




#690107 Tổng S = a + b + c = 0

Gửi bởi Nguyenhuyen_AG trong 10-08-2017 - 12:33

Làm vài cách khác đi anh

 

Trong ba số $a,\,b,\,c$ sẽ có hai số cùng dấu, giả sử đó là $a,\,b.$ Khi đó bất đẳng thức cần chứng minh tương đương với

\[\frac{b(b+c)}{c^2} + \frac{a^2}{b^2}+\frac{c^2}{a^2} + \frac{a}{b} + \frac{c}{a} \geqslant \frac{15}{4},\]

hay là

\[-\frac{ab}{(a+b)^2} + \frac{a^2}{b^2}+\frac{(a+b)^2}{a^2} + \frac{a}{b} - \frac{a+b}{a} \geqslant \frac{15}{4},\]

hoặc

\[-\frac{ab}{(a+b)^2} + \frac{a^2}{b^2}+\frac{b^2}{a^2} + \frac{a}{b} + \frac{b}{a} \geqslant \frac{15}{4},\]

\[\left[\frac{1}{4}-\frac{ab}{(a+b)^2}\right]+ \left(\frac{a^2}{b^2}+\frac{b^2}{a^2}-2\right)  + \left(\frac{a}{b} + \frac{b}{a}-2\right)  \geqslant 0.\]

Hiển nhiên đúng theo bất đẳng thức AM-GM.

 




#689922 Chứng minh rằng: $\frac{a+b+c}{3}\geq...

Gửi bởi Nguyenhuyen_AG trong 08-08-2017 - 20:20

Cho $a, b, c$ là các số thực dương thỏa $(a+b)(b+c)(c+a)=8.$ Chứng minh rằng: $\frac{a+b+c}{3}\geq \sqrt[27]{\frac{a^{3}+b^{3}+c^{3}}{3}}.$

 

Áp dụng bất đẳng thức cho $9$ số dương ta có

\[(a+b+c)^3 = a^3+b^3+c^3+3(a+b)(b+c)(c+a)=a^3+b^3+c^3+3\cdot 8 \geqslant 9\sqrt[9]{(a^3+b^3+c^3) \cdot 3^8}.\]

Từ đó suy ra điều phải chứng minh.




#689921 bất đẳng thức cauchy

Gửi bởi Nguyenhuyen_AG trong 08-08-2017 - 20:16

Sử dụng bất đẳng thức cô si tìm giá trị nhỏ nhât của S(x)= nx^(n+1)-(n+1)x^n+1

 

Viết bất đẳng thức trên lại như sau

\[\frac{n}{n+1} \cdot x^{n+1}+\frac1{n+1} \cdot 1 \geqslant x^n.\]

Chú ý rằng $\frac{n}{n+1}+\frac1{n+1}=1$ nên bất đẳng thức này đúng theo bất đẳng thức AM-GM suy rộng.




#689920 Bất đẳng thức lượng giác

Gửi bởi Nguyenhuyen_AG trong 08-08-2017 - 20:12

Cho tam giác ABC nhọn. CMR:

$\tan\frac{A}{2}+tan\frac{B}{2}+tan\frac{C}{2}\leq 2$

 

Ta có

\[\sum \tan\frac{A}{2} = \frac1p \sum r_a = \frac{4R+r}{p},\]

nên bất đẳng thức trên tương đương với

\[2(p-2R-r)+r \geqslant 0.\]

Hiển nhiên đúng do bất đẳng thức trong tam giác nhọn $p-2R-r \geqslant 0.$

 

Anh Huyện có một cách phản chứng khá hay: Giả sử bất đẳng thức trên là sai. Khi đó nó lại mâu thuẫn với yêu cầu bài toán là chứng minh bất đẳng thức trên đúng. Do đó bất đẳng thức trên đúng (dpcm).

 

Em spam ạ!

:lol:




#689919 Tổng S = a + b + c = 0

Gửi bởi Nguyenhuyen_AG trong 08-08-2017 - 19:49

Cho a , b , c đôi một khác nhau thỏa a + b + c = 0 . Cmr

$\sum \frac{a^{2}}{b^{2}} + \sum \frac{a}{b}\geqslant \frac{15}{4}$

 

Sao lại cần điều kiện đôi một khác nhau nhỉ?

 

Thay $c = -a-b$ bất đẳng thức trên trở thành

\[\frac{a^2}{b^2} + \frac{b^2}{(a+b)^2} + \frac{(a+b)^2}{a^2} + \frac{a}{b} - \frac{b}{a+b} - \frac{a+b}{a} \geqslant \frac{15}{4}.\]

Đặt $a=kb$ bất đẳng thức trên tương đương với

\[k^2 + \frac{1}{(k+1)^2} + \frac{(k+1)^2}{k^2} + k - \frac{1}{k+1} - \frac{k+1}{k} \geqslant \frac{15}{4},\]

hoặc

\[\frac{(k-1)^2(2k+1)^2(k+2)^2}{4k^2(k+1)^2} \geqslant 0.\]

Hiển nhiên đúng.